From e0c6872cf40896c7be36b11dcc744620f10adf1d Mon Sep 17 00:00:00 2001 From: Norbert Preining Date: Mon, 2 Sep 2019 13:46:59 +0900 Subject: Initial commit --- macros/latex/contrib/bhcexam/test2.tex | 144 +++++++++++++++++++++++++++++++++ 1 file changed, 144 insertions(+) create mode 100644 macros/latex/contrib/bhcexam/test2.tex (limited to 'macros/latex/contrib/bhcexam/test2.tex') diff --git a/macros/latex/contrib/bhcexam/test2.tex b/macros/latex/contrib/bhcexam/test2.tex new file mode 100644 index 0000000000..8dac567787 --- /dev/null +++ b/macros/latex/contrib/bhcexam/test2.tex @@ -0,0 +1,144 @@ +\documentclass[16kpaper]{BHCexam} +\begin{document} + +\maketitle +\mininotice + +\begin{questions} +\tiankong +\question 已知~$\bm{a}=(k,-9)$、$\bm{b}=(-1,k)$, $\bm{a}$~与~$\bm{b}$~为平行向量, + 则~$k=$\stk{$\pm3$}. + +\question 若函数~$f(x)=x^{6m^2-5m-4}\,(m\in\mathbb{Z})$~的图像关于~$y$~轴对称, + 且~$f(2)a>0$), 若再添加~$m\text{g}$~糖~($m>0$), + + +\question 已知~$f(x)=1-\textbf{c}_8^1x+\textbf{c}_8^2x^2-\textbf{c}_8^3x^3+\cdots+\textbf{c}_8^8x^8$, + 则~$f\big(\dfrac{1}{2}+\dfrac{\sqrt{3}}{2}\textbf{i}\big)$~的值是\ltk{$-\dfrac{1}{2}-\dfrac{\sqrt{3}}{2}\textbf{i}$}. + +\question 自然数~$1,2,3,\ldots,10$~的方差记为~$\sigma^2$, + 其中的偶数~$2,4,6,8,10$~的方差记为~$\sigma_1^2$, + 则~$\sigma^2$~与~$\sigma_1^2$~的大小关系为~$\sigma^2$\stk{$>$}$\sigma_1^2$. + +\question 若~$\theta$~为三角形的一个内角, 且~$\sin\theta+\cos\theta=\dfrac{2}{3}$, + 则方程~$x^2\csc\theta-y^2\sec\theta=1$~表示的曲线的焦点坐标是\stk{$\big(\pm\dfrac{\sqrt{6}}{3},0\big)$}. + + +\question 高为~$h$~的棱锥被平行于棱锥底面的截得棱台侧面积是 + 原棱锥的侧面积的~$\dfrac{5}{9}$, + 则截得的棱台的体积与原棱锥的体积之比是\stk{$19:27$}. + +\question 以椭圆~$\dfrac{x^2}{169}+\dfrac{y^2}{144}=1$~的右焦点为圆心, + 且与双曲线~$\dfrac{x^2}{9}-\dfrac{y^2}{16}=1$~的渐近线相切的圆方程是\mtk{$(x-5)^2+y^2=16$}. + + +\question 若~$\sqrt{\,\sin x}$~是有理数且~$x$~不是~$\dfrac{\pi}{6}$~的整数倍, + 则~$x$~可能取的值是\mtk{$\arcsin\dfrac{1}{4}$ 等}.(只要求写出一个) + +\question 马路上有编号~1~到~10~的~10~盏路灯, 为节约用电又不影响照明, + 可以关掉其中的~3~盏, 但又不能同时关掉相邻的两盏, 也不能关掉两端的路灯, + 满足条件的关灯方法有\stk{$20$}种. + + +\question 以椭圆~$\dfrac{x^2}{169}+\dfrac{y^2}{144}=1$~的右焦点为圆心, + 且与双曲线~$\dfrac{x^2}{9}-\dfrac{y^2}{16}=1$~的渐近线相切的圆方程是\mtk{$(x-5)^2+y^2=16$}. + +\question 若~$\sqrt{\,\sin x}$~是有理数且~$x$~不是~$\dfrac{\pi}{6}$~的整数倍, + 则~$x$~可能取的值是\mtk{$\arcsin\dfrac{1}{4}$ 等}.(只要求写出一个) + +\question 马路上有编号~1~到~10~的~10~盏路灯, 为节约用电又不影响照明, + 可以关掉其中的~3~盏, 但又不能同时关掉相邻的两盏, 也不能关掉两端的路灯, + 满足条件的关灯方法有\stk{$20$}种. + + +\question 以椭圆~$\dfrac{x^2}{169}+\dfrac{y^2}{144}=1$~的右焦点为圆心, + 且与双曲线~$\dfrac{x^2}{9}-\dfrac{y^2}{16}=1$~的渐近线相切的圆方程是\mtk{$(x-5)^2+y^2=16$}. + +\newpage + +\xuanze +\question 已知集合~$A=\{x\mid \abs{x-1}<3 \}$, +集合~$B=\{y| y=x^2+2x+1,x\in\mathbb{R}\}$, 则~$A\cap +\complement_U B$~为\stk{C}. +\twoch{$[\,0,4)$}{$(-\infty,-2\,]\cup[4,+\infty)$}{$(-2,0)$}{$(0,4)$} + +\question 若~$a$、$b$~是直线, $\alpha$、$\beta$~是平面, +则以下命题中真命题是\stk{D}.\\ +\fourch{若~$a$、$b$~异面, $a\subset\alpha$,$b\subset\beta$, 且~$a\perp b$, 则~$\alpha\perp\beta$}{若~$a\parallel b$, $a\subset\alpha$, $b\subset\beta$,则~$\alpha\parallel\beta$}{若~$a\parallel \alpha$, +$b\subset\beta$, 则~$a$、$b$ 异面}{若~$a\perp b$, $a\perp\alpha$,$b\perp\beta$, 则~$\alpha\perp\beta$} + +\question 已知集合~$A=\{x\mid \abs{x-1}<3 \}$, +集合~$B=\{y| y=x^2+2x+1,x\in\mathbb{R}\}$, 则~$A\cap +\complement_U B$~为\stk{C}. +\twoch{$[\,0,4)$}{$(-\infty,-2\,]\cup[4,+\infty)$}{$(-2,0)$}{$(0,4)$} + +\question 若~$a$、$b$~是直线, $\alpha$、$\beta$~是平面, +则以下命题中真命题是\stk{D}.\\ +\fourch{若~$a$、$b$~异面, $a\subset\alpha$,$b\subset\beta$, 且~$a\perp b$, 则~$\alpha\perp\beta$}{若~$a\parallel b$, $a\subset\alpha$, $b\subset\beta$,则~$\alpha\parallel\beta$}{若~$a\parallel \alpha$, +$b\subset\beta$, 则~$a$、$b$ 异面}{若~$a\perp b$, $a\perp\alpha$,$b\perp\beta$, 则~$\alpha\perp\beta$} + +\newpage +\jianda +\question 已知复数~$z$ 满足:$\abs{z}-z^*=\dfrac{10}{1-w\textbf{i}}$(其中~$z^*$ +是~$z$ 的共轭复数). +\begin{parts} +\part[7] 求复数~$z$; +\part[7] 若复数~$w=\cos\theta+\textbf{i}\sin\theta\,(\theta\in\mathbb{R})$, 求~$\abs{z-2}$ 的取值范围. +\end{parts} + +\begin{solution} +\begin{parts} +\part $z=3+4\textbf{i}$ +\part $\abs{z-w}\in[4,6]$ +\end{parts} +\end{solution} + +\question[14] 函数~$f(x)=4\sin\dfrac{\pi}{12}x\cdot\sin + \left(\dfrac{\pi}{2}+\dfrac{\pi}{12}x\right),x\in[a,a+1]$, + 其中常数~$a\in[0,5]$, 求函数~$f(x)$ 的最大值~$g(a)$. + +\begin{solution} +略 +\end{solution} + +\newpage + +\question[16] 函数~$f(x)=4\sin\dfrac{\pi}{12}x\cdot\sin + \left(\dfrac{\pi}{2}+\dfrac{\pi}{12}x\right),x\in[a,a+1]$, + 其中常数~$a\in[0,5]$, 求函数~$f(x)$ 的最大值~$g(a)$. + +\begin{solution} +略 +\end{solution} + +\newpage +\question 已知复数~$z$ 满足:$\abs{z}-z^*=\dfrac{10}{1-w\textbf{i}}$(其中~$z^*$ +是~$z$ 的共轭复数). +\begin{parts} +\part[8] 求复数~$z$; +\part[8] 若复数~$w=\cos\theta+\textbf{i}\sin\theta\,(\theta\in\mathbb{R})$, 求~$\abs{z-2}$ 的取值范围. +\end{parts} + +\begin{solution} +\begin{parts} +\part $z=3+4\textbf{i}$ +\part $\abs{z-w}\in[4,6]$ +\end{parts} +\end{solution} + +\newpage + +\question[18] 函数~$f(x)=4\sin\dfrac{\pi}{12}x\cdot\sin + \left(\dfrac{\pi}{2}+\dfrac{\pi}{12}x\right),x\in[a,a+1]$, + 其中常数~$a\in[0,5]$, 求函数~$f(x)$ 的最大值~$g(a)$. + +\begin{solution} +略 +\end{solution} + +\end{questions} +\end{document} -- cgit v1.2.3